LSAT and Law School Admissions Forum

Get expert LSAT preparation and law school admissions advice from PowerScore Test Preparation.

 sndhlpstt
  • Posts: 3
  • Joined: Oct 08, 2023
|
#103565
Can someone please explain why E is incorrect? Is it because it is not supported, only perhaps strengthens?
 Luke Haqq
PowerScore Staff
  • PowerScore Staff
  • Posts: 787
  • Joined: Apr 26, 2012
|
#103640
Hi sndhlpstt!

One thing worth noting about this stimulus is that there is no conclusion in it. This should be a big clue that one is dealing with a must-be-true question. The question stem confirms this, asking for what is "most strongly supported" by the stimulus.

Since there's no conclusion, there's nothing for (E) to strengthen. Rather, it's something that we don't know to be true based on the stimulus alone. It could be true, but we're asked for what we can know or infer must be true based on the stimulus.

We know that the film historians find it difficult to ascertain typical audience members' views of films in the early twentieth century, and we also know that "newspaper and magazine reviews fail to provide much insight." We can therefore infer that these reviews do not illuminate typical audience members' views of films.

Get the most out of your LSAT Prep Plus subscription.

Analyze and track your performance with our Testing and Analytics Package.